Calculus 10th Edition

Published by Brooks Cole
ISBN 10: 1-28505-709-0
ISBN 13: 978-1-28505-709-5

Chapter 9 - Infinite Series - 9.2 Exercises - Page 601: 10

Answer

The series diverges

Work Step by Step

$if\lim _{n\rightarrow \infty }a_{n}\neq 0\Rightarrow \sum ^{n}_{1}a_{n}$ diverges $\lim _{n\rightarrow \infty }\dfrac {\eta }{2n+3}=\dfrac {n/n}{2n/n+3/n}=\dfrac {1}{2+\dfrac {3}{n}}=\dfrac {1}{2}\neq 0$ So the series diverges
Update this answer!

You can help us out by revising, improving and updating this answer.

Update this answer

After you claim an answer you’ll have 24 hours to send in a draft. An editor will review the submission and either publish your submission or provide feedback.